LSAT and Law School Admissions Forum

Get expert LSAT preparation and law school admissions advice from PowerScore Test Preparation.

 Administrator
PowerScore Staff
  • PowerScore Staff
  • Posts: 8917
  • Joined: Feb 02, 2011
|
#23172
Complete Question Explanation

Flaw in the Reasoning. The correct answer choice is (C)

In this stimulus, Yolanda presents an argument and then Arjun refutes it by presenting an example. Yolanda presents similarities between computer manipulation and joyriding but then points out a difference between them. Arjun disagrees with that difference by presenting a hypothetical situation. Arjun's conclusion is that "computer crimes also cause physical harm to people." However, he only presents a hypothetical situation and does not show that these crimes actually cause harm, as he asserts. This is the flaw in his argument.

Answer choice (A) Arjun tries to demonstrate that the distinction made in Yolanda's argument is inaccurate, so this answer choice is not a flaw in his reasoning but what his argument tries (but fails) to accomplish.

Answer choice (B) Although Arjun does deny Yolanda's conclusion, he also does provide evidence, i.e. his hypothetical situation.

Answer choice (C): This is the correct answer choice. For Arjun's conclusion to be true, computer crimes must cause harm to people. He has only shown that they could possibly cause harm in one hypothetical circumstance; he has not shown that they do in reality.

Answer choice (D) This answer choice employs the word "necessary" to try to tempt you, but does not apply to Arjun's argument in any way.

Answer choice (E) Arjun's evidence is consistent with his conclusion, so this is not the correct answer.
 sna1
  • Posts: 1
  • Joined: Aug 29, 2015
|
#19580
I'm not sure what the name of the error of reasoning in the answer choice is.

What is the error of reasoning displayed by using the idea that something may happen as evidence that it will happen
 Jon Denning
PowerScore Staff
  • PowerScore Staff
  • Posts: 904
  • Joined: Apr 11, 2011
|
#19584
Hey sna,

Thanks for the question and welcome to the Forum! Since you've mentioned page 7-92 I gather that you're a PowerScore student (congrats!), which is great because it means I can cross-reference other pages/elements in the Flaw section of your course book as we work through this questions.

That in mind, let me start by saying that the majority of Flaw stimuli you'll encounter on the LSAT will exhibit one of the categories of flaw discussed during class for Lesson 7 (covered in the lesson portion of your book), but not all will. That is, those categories—Source, Circular, Evidence, Composition/Division, etc.—come up a lot, as both the actual flaw as well as the incorrect answers, but reasoning can be flawed for less well-defined or easily-categorized reasons, too.

So I'd encourage you to approach Flaw questions with the intention of classifying flaws according to known types, but with an open-mindedness to other, unclassified mistakes on the part of the author. The test makers will undoubtedly present you with both.

For #38, the error in Arjun's argument comes from the logical leap he makes from stating that something "could" happen (unauthorized use could damage life-dependent data systems), to the conclusion that it will happen (unauthorized use will cause physical harm to people). As (C) states, he mistakenly relies on the actuality/certainty of a phenomenon when all that he has shown is that it's possible.

Categorically-speaking I'd say this is a type of Evidence Error, where Arjun has taken evidence that his position could be true as evidence that it is in fact true. This is discussed in some detail on page 7-17 under point #4. But again be reminded that these categories are to be used as a reference guide rather than an exhaustive list :)

I hope that helps and keep up the hard work! You're in the home stretch now!

Jon
 kcho10
  • Posts: 69
  • Joined: Nov 02, 2015
|
#26673
Hello,

Based on question #13, can't the 'I disagree!' in Arjun's argument be inferred to refer to Yolanda's claim that joyriding is more dangerous than recklessly using private property?

If so, Arjun is refuting Yolanda's conclusion that joyriding is MORE dangerous. But Arjun only provides evidence that reckless use of private property is also dangerous. This doesn't provide evidence that one is not more dangerous than the other. So why can't (B) be correct?

Is it because even if irrelevant evidence is provided to deny an argument, it is still considered evidence against an argument?

Thank you in advance
 Shannon Parker
PowerScore Staff
  • PowerScore Staff
  • Posts: 147
  • Joined: Jun 08, 2016
|
#26684
Hi,

I understand what you are getting at, but i think here, that providing evidence that it is dangerous is providing evidence against the conclusion that joyriding is more dangerous. It may not be sufficient evidence, but it any evidence of how dangerous computer crimes are is some small evidence against joyriding being more dangerous. Thus ruling out (B).

The key is to realize that his example has the word could in it, and there is an answer choice specifically addressing the fact that he uses an example which is so far only a possibility. (C).

Hope this helps

~Shannon
 kcho10
  • Posts: 69
  • Joined: Nov 02, 2015
|
#26690
Shannon Parker wrote:Hi,

I understand what you are getting at, but i think here, that providing evidence that it is dangerous is providing evidence against the conclusion that joyriding is more dangerous. It may not be sufficient evidence, but it any evidence of how dangerous computer crimes are is some small evidence against joyriding being more dangerous. Thus ruling out (B).

The key is to realize that his example has the word could in it, and there is an answer choice specifically addressing the fact that he uses an example which is so far only a possibility. (C).

Hope this helps

~Shannon
this helps out a lot. Thank you so much!
 Blueballoon5%
  • Posts: 156
  • Joined: Jul 13, 2015
|
#45595
Could someone help me understand why answer choice D is wrong? When I read this choice, I thought that Arjun mistakes this hypothetical situation as a necessary element to his overall conclusion that Yolanda's argument is wrong (the argument being joyriding is more dangerous than computer crimes).
 Adam Tyson
PowerScore Staff
  • PowerScore Staff
  • Posts: 5153
  • Joined: Apr 14, 2011
|
#49595
Answer D is interesting in that it presents a mix of conditional and causal reasoning. If Arjun "(mistook) something that leads to his conclusion," that would mean something actually caused his conclusion to be true. His conclusion is that computer crimes do cause physical harm. What caused that? Nothing, actually - he wants us to believe it is true, but there is no evidence that something actually causes it to be true. Next in answer D we have "for something that is necessary for his conclusion." Does Arjun think his conclusion - that computer crimes cause physical harm - is sufficient for some other claim? I'm not seeing that conditional relationship in his argument.

Mainly, we can reject this answer choice because neither Arjun's argument nor his flaw are conditional in nature. The problem is that he treated a "could" premise as a "must" premise. In other words, this is an evidence flaw, treating some evidence that supports a particular conclusion as if that evidence actually proves that conclusion is true. Before we pick a conditional answer to a flaw question we need to be sure that the argument is in fact conditional! Otherwise, answers like that are just there to confuse and distract us. The same holds true for a causal answer to a flaw question - if the argument isn't causal, then the flaw won't be described in causal terms. This argument involved causation, but it wasn't a causal flaw, because they author didn't argue that two things are correlated and therefore one must have caused the other. Instead it was "it could cause harm, so it does cause harm."
User avatar
 abby1285
  • Posts: 11
  • Joined: Mar 20, 2021
|
#85732
Hi! Could someone explain why A is wrong? The way I read the argument was that Yolanda is asserting that joyriding is the "more dangerous crime" and Arjun disputes by saying no, computer crimes could also be dangerous. I took the error of reasoning to be that Arjun is failing to understand Yolanda's argument that joyriding is the MORE dangerous crime. Thus, I picked A because Arjun failed to maintain the distinction (joyriding = more dangerous) in Yolanda's argument. I originally kept C as a contender but thought that it didn't really matter whether or not the danger is hypothetical. Thanks!
User avatar
 KelseyWoods
PowerScore Staff
  • PowerScore Staff
  • Posts: 1079
  • Joined: Jun 26, 2013
|
#85765
Hi Abby!

Yolanda's conclusion is that joyriding is more dangerous but she also says that only intellectual property is harmed in the case of computer crimes. So she is not saying that both of them harm people and joyriding is just more dangerous. She argues that computer crimes do not harm people. So Arjun is not failing to understand Yolanda's argument. He is taking issue with the very last part of her argument about only intellectual property being harmed in the case of computer crimes.

The distinction in Yolanda's argument is that joyriding is more dangerous because it harms people and that computer crimes harm only intellectual property. She's making a distinction between joyriding and computer crimes based on who/what they harm. Arjun does not fail to maintain that distinction. He just disagrees with that distinction because he thinks that computer crimes can also harm people. Attacking the distinction is not "failing to maintain it" and it is also not a flaw. If his point is that her distinction is incorrect, that's fine. But he needs to provide proper support to prove his claim that her distinction is incorrect.

Flaws always come down to the specific conclusion that is being made and the specific premises that are being used to support that conclusion. Ask yourself: 1) What is the conclusion?; 2) What are the premises?; and 3) Why don't the premises given fully prove the conclusion as stated? Arjun's conclusion is "computer crimes also cause physical harm to people" but his premise is just that it is possible for computer crimes to cause harm to people. He has not proven that they actually do cause harm to people.

Hope this helps!

Best,
Kelsey

Get the most out of your LSAT Prep Plus subscription.

Analyze and track your performance with our Testing and Analytics Package.